Prove that a differential function is bounded by 1/2

Click For Summary
The discussion revolves around proving that a function φ(x) with a continuous derivative, satisfying the condition φ'(x) + 2φ(x) ≤ 1 and φ(0) = 0, is bounded by 1/2 for x ≥ 0. An initial attempt involved manipulating the differential equation using an integrating factor, leading to the expression e^(2x)φ(x) ≤ e^(2x). A suggestion was made to integrate the derived inequality from 0 to x, which clarified the approach. This integration ultimately leads to the conclusion that φ(x) must indeed be less than 1/2 for all x in the specified range. The discussion highlights the importance of proper integration techniques in solving differential inequalities.
DeadOriginal
Messages
274
Reaction score
2

Homework Statement


Suppose ##\phi(x)## is a function with a continuous derivative on ##0\leq x<\infty## such that ##\phi'(x)+2\phi(x)\leq 1## for all such ##x## and ##\phi(0)=0##. Show that ##\phi(x)<\frac{1}{2}## for ##x\geq 0##.


The Attempt at a Solution


I tried to solve this like I would any other first order differential equation.
$$
\phi'(x)+2\phi(x)\leq 1\Leftrightarrow e^{2x}(\phi'(x)+2\phi(x))\leq e^{2x}\Leftrightarrow e^{2x}\phi(x)\leq e^{2x}
$$
so
$$
\phi(x)\leq e^{-2x}\int\limits_{x_{0}}^{x}e^{2t}dt + ce^{-2x}=\frac{1}{2}+ce^{-2x}
$$
but that was as far as I could get. Any help would be greatly appreciated.
 
Physics news on Phys.org
DeadOriginal said:

Homework Statement


Suppose ##\phi(x)## is a function with a continuous derivative on ##0\leq x<\infty## such that ##\phi'(x)+2\phi(x)\leq 1## for all such ##x## and ##\phi(0)=0##. Show that ##\phi(x)<\frac{1}{2}## for ##x\geq 0##.


The Attempt at a Solution


I tried to solve this like I would any other first order differential equation.
$$
\phi'(x)+2\phi(x)\leq 1\Leftrightarrow e^{2x}(\phi'(x)+2\phi(x))\leq e^{2x}\Leftrightarrow e^{2x}\phi(x)\leq e^{2x}
$$
so
$$
\phi(x)\leq e^{-2x}\int\limits_{x_{0}}^{x}e^{2t}dt + ce^{-2x}=\frac{1}{2}+ce^{-2x}
$$
but that was as far as I could get. Any help would be greatly appreciated.

You almost have it. After you multiply by your integrating factor you have$$
(e^{2x}\phi(x))'\le e^{2x}$$Instead of doing an indefinite integral, integrate this from ##0## to ##x##:$$
\int_0^x(e^{2t}\phi(t))'~dt\le \int_0^x e^{2t}~dt$$and see what happens.
 
Ahh! I see it! Thanks!
 
Question: A clock's minute hand has length 4 and its hour hand has length 3. What is the distance between the tips at the moment when it is increasing most rapidly?(Putnam Exam Question) Answer: Making assumption that both the hands moves at constant angular velocities, the answer is ## \sqrt{7} .## But don't you think this assumption is somewhat doubtful and wrong?

Similar threads

  • · Replies 105 ·
4
Replies
105
Views
6K
  • · Replies 1 ·
Replies
1
Views
1K
  • · Replies 13 ·
Replies
13
Views
2K
  • · Replies 6 ·
Replies
6
Views
2K
  • · Replies 4 ·
Replies
4
Views
1K
  • · Replies 14 ·
Replies
14
Views
2K
  • · Replies 4 ·
Replies
4
Views
2K
  • · Replies 5 ·
Replies
5
Views
2K
  • · Replies 13 ·
Replies
13
Views
4K